scientific notation5.1x10⁶ x 2.3x10⁶

Answers

Answer 1

The given expression is

5.1 x 10^6 x 2.3 x 10^6

We would apply the law of exponents which is expressed as

a^b x a^c = a^(b + c)

By applying this, we have

5.1 x 2.3 x 10^6 x 10^6

= 11.73 x 10^(6 + 6)

= 11.73 x 10^12


Related Questions

Question 4 of 10Select the correct product of the exponential expression.35A. 5.5.5B. 3.5C. 15D. 3.3.3.3.3SUBMIT

Answers

You have the following expression:

[tex]3^5[/tex]

it means that the number 3 is multipled by itself five times (because of the exponent), then, the previous expression can be written as follow:

[tex]3^5=3\cdot3\cdot3\cdot3\cdot3[/tex]

3/4 divided by 2/3 using fractions

Answers

To solve this divition we can write it like:

[tex]\frac{\frac{3}{4}}{\frac{2}{3}}[/tex]

So now we can multiplicate the numerator of the first fraction with the denominatior of the second fraction, and put the resould in the numerator of the quationt, ans the same operation for the numeratior with the numerator of the secon fraction and the denominator of the first fraction so:

[tex]\frac{3\cdot3}{4\cdot2}=\frac{9}{8}[/tex]

given: s is the midpoint of QR , QR , PS and angle RSP and angle QSP are right angles prove PR is congruent to PQ

Answers

∆RSP ≈ ∆QSP through SAS congruency theorem. PR is congruent to PQ.

Given that,

S being the midpoint of QR

SR = QS            (∵ midpoint)

PS = PS  (common side/reflexive property)

Two right triangles are congruent due to

∵ PS ≅ PS                       (SAS congruency)

According to the SAS rule, two triangles are said to be congruent if any two sides and any angle contained between the sides of one triangle are equal to the second triangle's two sides and angle between its sides correspond to the first triangle's.

QS ≅ PS

Thus, ∆RSP ≈ ∆QSP through SAS congruency.

While PQ = PR               (by CPCT).

Hence, proved.

Therefore, ∆RSP ≈ ∆QSP through SAS congruency theorem. PR is congruent to PQ.

To learn more about congruency visit: https://brainly.com/question/7888063

#SPJ9

The park near Amber's house has a path around its perimeter 3 that is mile long. Amber's goal is to walk 4.5 miles a day. If 4 Amber reaches her daily goal, how many times will Amber walk around the park?

Answers

We know that

• The path is 3 miles long.

,

• Amber's goal is 4.5 miles a day.

To find the number of times she will walk around the park, we have to divide.

[tex]\frac{4.5}{3}=1.5[/tex]Hence, Amber will walk around 1 entire lap and a half.

Greetings, i need help with this math problem. Thank you

Answers

The numerator of the left hand side can be rewritten as:

[tex]x^2+6x+9=(x+3)^2[/tex]

Then, the given equation can be written as:

[tex]\frac{(x+3)^2}{x+3}=0[/tex]

Since

[tex](x+3)^2=(x+3)(x+3)[/tex]

we have

[tex]\frac{(x+3)(x+3)}{x+3}=0[/tex]

We can to can cancel out one term x+3 and get

[tex]x+3=0[/tex]

which gives

[tex]x=-3[/tex]

Finally, in order to check that this value corresponds to a real answer, we need to subsitute this value into the equation and compute the limit when x approaches to -3, that is,

[tex]lim_{x\rightarrow-3}\frac{x^2+6x+9}{x+3}[/tex]

which gives

[tex]l\imaginaryI m_{x\operatorname{\rightarrow}-3}\frac{x^{2}+6x+9}{x+3}=\frac{0}{0}[/tex]

Since the limit has the form 0/0 we can to apply L'Hopital rule, that is,

[tex]l\imaginaryI m_{x\operatorname{\rightarrow}-3}\frac{\frac{d}{dx}(x^2+6x+9)}{\frac{d}{dx}(x+3)}[/tex]

which gives

[tex]l\imaginaryI m_{x\operatorname{\rightarrow}-3}\frac{\frac{d}{dx}(x^{2}+6x+9)}{\frac{d}{dx}(x+3)}=l\imaginaryI m_{x\operatorname{\rightarrow}-3}\frac{2x+6}{1}=\frac{0}{1}=0[/tex]

Since the limit exists and is equal to zero then the solution of the equation is: x= -3

graph a piecewise function with 3 equations and sketch a graph

Answers

Solution:

Given:

[tex]h(x)=\begin{cases}2x,x\le-2 \\ x^2-1,-2A piecewise function is a function that is defined by different formulas or functions for each given interval.

It is a function in which more than one formula is used to define the output over different pieces of the domain.

The function h(x) given has three outputs for three different domains.

[tex]\begin{gathered} \text{The first is a linear function;} \\ h(x)=2x \\ \\ \text{The second is a quadratic function;} \\ h(x)=x^2-1 \\ \\ \text{The third is a linear function;} \\ h(x)=x-3 \end{gathered}[/tex]

Therefore, the graph using a graph plotter (desmos) is as shown below;

A survey asked "Do you think the president is doing a great job?" Of the 1200 Americans surveyed, 800 responded yes. For a 95% level of confidence find the sample proportion and the margin of error associated with the poll.

Answers

We have here working with estimating a population proportion.

We have the following information from the question:

• The sample size, n, is equal to 1200 (n = 1200).

,

• We have that the fraction that responded "Yes" is 800.

,

• We need to find a 95% level of confidence for the margin error associated with the poll.

Now, we have the sample proportion for the sample size, n = 1200 is as follows:

Sample Proportion[tex]\begin{gathered} \hat{p}=\frac{800}{1200}=\frac{2}{3}\approx0.666666666667\approx0.67 \\ \\ \hat{p}=\frac{800}{1200}\approx0.67 \end{gathered}[/tex]

Therefore, the sample proportion for the sample size is 800/1200, which is, approximately, 0.67.

The margin of error associated with the poll

The margin of error, in this case, is given by the next formula:

[tex]E=Z_c\sqrt{\frac{\hat{p}(1-\hat{p})}{n}}[/tex]

Where:

[tex]\begin{gathered} Z_c\text{ is the critical value for a 95\% level of confidence} \\ \\ n\text{ is the sample size \lparen n = 1200\rparen} \\ \\ \hat{p}\text{ is the sample proportion \lparen800/1200\rparen} \\ \end{gathered}[/tex]

Now, we have that, for a level of confidence of 95%, the critical value is equal to z = 1.96:

Now, using all of the values at our disposal, we can use the formula to find the margin of error as follows:

[tex]\begin{gathered} E=Z_c\sqrt{\frac{\hat{p}(1-\hat{p})}{n}} \\ \\ E=1.96\sqrt{\frac{\frac{8}{12}(1-\frac{8}{12})}{1200}} \\ \\ E=0.0266722216436\approx0.027 \end{gathered}[/tex][tex]\begin{gathered} E=Z_c\sqrt{\frac{\hat{p}(1-\hat{p})}{n}} \\ \\ E=1.96\sqrt{\frac{\frac{8}{12}(1-\frac{8}{12})}{1200}} \\ \\ E=0.0266722216436\approx0.027 \end{gathered}[/tex]

Therefore, in summary, we have that:

1. The sample proportion is:

[tex]\hat{p}=\frac{800}{1,200}\approx0.67[/tex]

2. The margin of error associated with the poll is:

[tex]E=0.0266722216436\approx0.027[/tex]

A multiple choice test contains 10 questions with 5 answer choices. What is the probability of correctly answering 5 questions if you guess randomly on each question?A. 0.9936B. 0.2C. 0.0264D. 0.0003

Answers

If there are 10 questions with 5 answer choices, then first we need to find out the probability of getting the first questions randomly correct.

Therefore, that is:

[tex]\begin{gathered} Probability\text{ of getting 1 answer correct= }\frac{1}{5} \\ \\ Then\text{ if we need to get the second question correct it is:} \\ \frac{1}{5}(first\text{ question\rparen x }\frac{1}{5}=\text{ \lparen}\frac{1}{5})^2 \\ \\ And\text{ for the other questions applies the same. Therefore, if we need 5 correct answers, then:} \\ \frac{1}{5}\text{ x }\frac{1}{5}x\frac{1}{5\frac{}{}}x\frac{1}{5}x\text{ }\frac{1}{5}=\text{ \lparen}\frac{1}{5})^5\text{ = }\frac{1}{3125}=\text{ 0.0003} \end{gathered}[/tex]

The answer is D. 0.0003

And each diagram below, right, the 2 number on the sides of the acts that are multiplied together to get the top number of the x, but added together to get the bottom number of the x.

Answers

-34 = x*y

15 = x +y

17 and -2

__________________________

_____________

9 = x*y

6 = x +y

_

There are 38 coins in a collection of 20 paise coins and 25 paise coins. If the total value of the collection is Rupees 8.50, how many of each are there?

Answers

We will have the following:

*First:

**We stablish that x will represent the number of 20 paise coins.

**We stablish that y will represent the number of 25 paise coins.

Second: From this we will then have:

[tex]x+y=38[/tex]

&

[tex]20x+25y=850[/tex]

[This 850 is due to the fact that 8.50 Rupees are equal to 850 paise].

*Third: We solve for either x or y in the first equation:

[tex]x=38-y[/tex]

Now, we replace this in the second equation and solve for y:

[tex]20(38-y)+25y=850\Rightarrow760-20y+25y=850[/tex][tex]\Rightarrow5y=90\Rightarrow y=18[/tex]

So, we have that there are 18 25 paise coins.

Now, using this we solve for x in the first equation:

[tex]x+18=38\Rightarrow x=20[/tex]

So, we have that there are 20 20 pais coins.

ANYONE HELP ME WITH THE AREA OF THE FLOOR PLAN FOR THE OFFICE
NEED CLEAR EXPLAINATION AND ANSWER.

Answers

The area of the floor plan for the office is 2200 m².

We are given a diagram. The diagram shows the floor plan of an office. The height of the office is 50 meters. The length of the floor of the office is 55 meters. The width of the floor of the office is 40 meters. We need to find the area of the floor of the office.

The shape of the floor of the office is a rectangle. The area of a rectangle is calculated as the product of its length and width. Let the area of the floor of the office be represented by the variable "A".

A = 55*40

A = 2200

Hence, the area of the floor of the office is 2200 m².

To learn more about area, visit :

https://brainly.com/question/27683633

#SPJ1

Find the area of quadrilateral ABCD. Round the area to the nearest whole number, if necessary.у| A(-5,4)4B(0, 3)22F(-2,1)-226 xTC(4, -1)-4E(2, -3)D(4, -5)6The area issquare units.

Answers

We have a quadrilateral ABCD and we want to calculate the area.

We can divide it in three areas (two triangles and one rectangle) and then add the surfaces.

As it is rotated 45 degrees, we can define a "new unit" that is the diagonal of a square of 1 by 1 unit, in the scale of the graph.

This new unit, the diagonal that we will call "d", by the Pythagorean theorem, has a value of:

[tex]d=\sqrt{2}[/tex]

We will start then with the triangle ABF. It has a side BF that has a value of 2 diagonals (2d) and a side FA that has a value of 3 diagonals (3d). The area of a triangle is half the multiplication of this two sides, so we have:

[tex]\frac{\bar{BF}\cdot\bar{FA}}{2}=\frac{2d\cdot3d}{2}=3d^2=3(\sqrt{2})^2=3\cdot2=6[/tex]

The second triangle is CED. We repeat the process and we have:

[tex]\frac{\bar{CE}\cdot\bar{ED}}{2}=\frac{2d\cdot2d}{2}=2d^2=2\cdot2=4[/tex]

The rectangle BCEF has an area of:

[tex]\bar{BF}\cdot\bar{EF}=2d\cdot4d=8d^2=8\cdot2=16[/tex]

Now we have the three areas. If we add them we get the area of ABCD:

[tex]6+4+16=26[/tex]

The quadrilateral ABCD has an area of 26 units^2.

Which of the following equations does the graph below represent?

A. -3x - 6y = 36

B. -3x + 6y = 36

C. x + 6y = 36

D. 3x + 6y = 36

Answers

The equation of the line which represents the given graph is -3x + 6y = 36

We can infer from the graph that the equation of the line passes through the points (0,6) and (-12,0).

Therefore the slope of the line passing through these two points is given by:

m = (0-6)/(-12-0)

or, m = 1 / 2

Now we can use the general equation of the line to get the required equation.

y-6 = 1/2 (x-0)

or, 2y - x = 12

Now we will multiply throughout by 3 we get:

6y - 3x = 36

or, -3x + 6y = 36

The general equation for a straight line is y = mx + c, where m stands for the line's slope and c for its y-intercept. The most frequently used straight line equation is one that has to do with geometry.

There are numerous ways to express the equation of a straight line, such as slope-intercept form,  point-slope form, standard form, general form,  etc.

Hence the required equation is -3x + 6y = 36 .

To learn more about equation visit:

https://brainly.com/question/2564656

#SPJ1

How is the series 7 + 13 + 19+...+ 139 represented in summation notation?

Answers

Each term is 6 greater than the previous term.

First term is "7".

So,

a = 7

d = 6

Let's find the formula for the series,

[tex]\begin{gathered} a+(n-1)d \\ 7+(n-1)(6) \\ 7+6n-6 \\ 6n+1 \end{gathered}[/tex]

We can immediately eliminate the firsst and third choice.

The variable is "t", so the general formula will be:

[tex]6t+1[/tex]

How many terms are there?

The series starts from t = 1,

since 6(1) + 1 = 6 + 1 = 7

and 6(2) + 1 = 12 + 1 = 13

The terms match!

So, 2nd answer choice is correct!!

Answer[tex]\sum ^{23}_{t\mathop=1}(6t+1)[/tex]

A regular polygon with 9 sides (a nonagon) has a perimeter of 72 inches. What is the area of this polygon? Provide mathematical evidence.

Answers

The perimeter of the regular nonagon is 72 inches, the length of each side can be determined as,

[tex]\begin{gathered} P=9s \\ 72=9s \\ s=8\text{ inches} \end{gathered}[/tex]

The diagram can be drawn as,

The value of apopthem a can be determined as, where n is the number of sides,

[tex]\begin{gathered} a=\frac{s}{2\tan (\frac{180^{\circ}}{n})} \\ =\frac{8}{2\tan20^{\circ}} \\ =10.98\text{ in} \end{gathered}[/tex]

The area can be determined as,

[tex]\begin{gathered} A=\frac{P\times a}{2} \\ =\frac{72\text{ inches}\times10.98\text{ inches}}{2} \\ =395.63in^2 \end{gathered}[/tex]

Thus, the required area of the polygon is 395.63 square inches.

make an equation to find the area of rectangle. move number and symbols to the line

Answers

Area of a rectangle

We know that the multiplication of both sides of the rectangle is the area of it.

In this case

Area: 6 x 8 = 48

Z + 24 = -33one step equation

Answers

We solve as follows:

[tex]z=-57[/tex]

We operate like terms after substracting 24 from each side of the function.

A farmer has 36 ft of fencing and wants to enclose the maximum rectangular area for his llamas. Find the dimensions of three possible areas he could enclose. What do you think the maximum area is? Why?

Answers

The farmer would like to eclose a rectangle

First we know that the perimeter of the rectangle is

[tex]P=2x+2y[/tex]

We also know that we have 36 ft of fence, that is we only can enclose a rectangle of 36 ft of perimeter. Then

[tex]36=2x+2y[/tex]

From this we can find y

[tex]\begin{gathered} 36=2x+2y \\ 36-2x=2y \\ y=\frac{36-2x}{2} \\ y=18-x \end{gathered}[/tex]

The area of the rectangle is

[tex]A=xy[/tex]

But we know the value of y, plugging this value into the last equation we have that

[tex]A=x(18-x)[/tex]

To find three possible values for the area we only have to give values to x. This values have to be positive (since we can't have a negative lenght). We also notice that the value can't exceed 18 since that would mean a zero area. With those point in consideration we choose three values between zero and 18.

If x=3, then the area is

[tex]\begin{gathered} A=3(18-3) \\ =3(15) \\ =45 \end{gathered}[/tex]

If x=9, then the area is

[tex]\begin{gathered} A=9(18-9) \\ =9(9) \\ =81 \end{gathered}[/tex]

if x=15, then the area is

[tex]\begin{gathered} A=15(18-15) \\ =15(3) \\ =45 \end{gathered}[/tex]

Then we have three possible areas for the rectangle.

The maximum value for the area is 81, and we see that because the equation for the area is a parabola that opens down with vertex in the point (9,81)

Hi can someone please give a full explanation on how to solve this problem? I’ll give Brainliest

Answers

Answer:

C is correct.

Step-by-step explanation:

The area of the rectangle is 10 × 6, or 60 square meters.

The area of the triangle is 1/2 × 7 × 6, or 21 square meters.

So the area of the shaded region is the area of the rectangle minus the area of the triangle. That area is 60 - 21, or 39 square meters. C is correct.

f(x) = {(7,3), (5,3), (9,8).(11,4)}g(x) = {(5, 7),(3,5), (7,9), (9,11)}a) f-1(x)b) g-1(x)

Answers

Given the coordinates of the function;

f(x) = {(7,3), (5,3), (9,8).(11,4)}

The inverse of any coordinate points is gotten by changing the y coordinate valuewith the x coordinate values i.e y = x

f-1(x) = {(3,7), (3,5), (8,9), (4,11)

You can see that the x coordinates has been swapped with the y coordinates.

Similarly;

g-1(x) = {(7, 5),(5,3), (9,7), (11,9)}

A boy at an amusement park has 65 ride tickets. Each ride on the roller coaster costs 7 tickets. After riding the roller coaster as many times as he can, how many tickets will the boy have left?

Answers

ANSWER

[tex]32\text{ tickets}[/tex]

EXPLANATION

The group consists of 4 friends and each friend has 12 tickets.

Each friend uses 4 tickets to ride the roller coaster.

To find the number of tickets each friend has after the ride, subtract the number of tickets used for the ride from the number of tickets each friend had initially.

That is:

[tex]\begin{gathered} \Rightarrow12-4 \\ 8 \end{gathered}[/tex]

Now, to find the number of tickets the group has, multiply the number of friends in the group by the number of tickets left:

[tex]\begin{gathered} 4\cdot8 \\ 32\text{ tickets} \end{gathered}[/tex]

In ΔIJK, k = 53 cm, j = 66 cm and ∠J=64°. Find all possible values of ∠K, to the nearest 10th of a degree.

Answers

The value of ∠K is 46.2° as the definition of angle is "An angle is created by joining two line segments at one point, or we can say that an angle is the combination of two line segments at a common endpoint".

What is angle?

An angle is created by joining two line segments at one point, or we can say that an angle is the combination of two line segments at a common endpoint. When two straight lines or rays intersect at a single endpoint, an angle is created. The vertex of an angle is the location where two points come together. The Latin word "angulus," which means "corner," is where the word "angle" originates. Based on measurement, there are different kinds of angles in geometry. The names of fundamental angles include acute, obtuse, right, straight, reflex, and full rotation. A geometrical shape called an angle is created by joining two rays at their termini. In most cases, an angle is expressed in degrees.

Here,

Side i = 68.91521 cm

Side j = 66 cm

Side k = 53 cm

Angle ∠I = 69.8°

Angle ∠J = 64°

Angle ∠K = 46.2°

∠K= sin⁻¹(k·sin(J)/j)

=sin⁻¹(53.(sin 64°)/66)

=46.2°

Since the definition of an angle is "An angle is created by joining two line segments at one point, or we can say that an angle is the combination of two line segments at a common endpoint," the value of ∠K is 46.2°.

To know more about angles,

https://brainly.com/question/28451077?referrer=searchResults

#SPJ1

The total number of photos on Hannah's camera is a linear function of how long she was in Rome. She already had 44 photos on her camera when she arrived in Rome. Then she took 24 photos each day for 6 days. What is the initial value of the linear function that represents this situation? 24 photos 44 photos 6 days o days per day

Answers

she already had 44 photos

She took 24 photos each day for 6 days.

The function is:

y= 44+24x

where y is the total number of photos and x is the number of days.

The initial value is when x=0

y= 44+24(0) =44

The initial value is 44.

A) 9- (-22) - 13B) 9 - (-22) = -13A football team gained 9yards on one play andthen lost 22 yards on thenext. Write a sum ofintegers to find theoverall change in fieldposition-9- (-22) = -13D) None of the above

Answers

The sum of integers should be written like:

9 + (-22) = -13

because the are asking in fact for a SUM of integers

The two integers are 9 and -22

when added you get:

9 + (-22) = 9 - 22 = -13

Latoya cut a circle into & equal sections and arranged the pieces to form a shape resembling a parallelogram. So in of

Answers

Based on the diagram, the base length of the new shape is half the circumference of the circle as indicated by 1/2C.

If f(x) = 2x^3 + 10x^2 + 18x + 10 and x + 1 is a factor of f(x), then find all of the zeros of f(x) algebraically

Answers

Given the polynomial:

[tex]f(x)=2x^3+10x^2+18x+10[/tex]

We know that (x + 1) is a factor of f(x). We divide f(x) by (x + 1):

Then:

[tex]f(x)=(x+1)(2x^2+8x+10)=2(x+1)(x^2+4x+5)[/tex]

For the quadratic term, we solve the following equation:

[tex]x^2+4x+5=0[/tex]

Using the general solution for quadratic equations:

[tex]\begin{gathered} x=\frac{-4\pm\sqrt{4^2-4\cdot1\cdot5}}{2\cdot1}=\frac{-4\pm\sqrt{16-20}}{2}=\frac{-4\pm\sqrt{4}}{2} \\ \\ \therefore x=-2\pm i \end{gathered}[/tex]

The zeros of f(x) are:

[tex]\begin{gathered} x_1=-1 \\ \\ x_2=-2-i \\ \\ x_3=-2+i \end{gathered}[/tex]

i don't know how to identify the domain and range of the graph

Answers

The domain is the set of all possible values for x. All x values ( horizontal axis) that are going to be used

Domain = (-4,-1,0,4)

The range is the set of all possible y-values . All the y-values (vertical axis) that are used.

Range = (-5)

Find the area of a rectangle that is 3 3 over 4 inches long by 2 1 fourth inches wide. ANS.( Use mixed number) _______. in squared

Answers

Let's begin by listing out the given information:

[tex]\begin{gathered} Length(l)=3\frac{3}{4} \\ Width(w)=2\frac{1}{4} \\ Area=l\cdot w \\ Area=3\frac{3}{4}\cdot2\frac{1}{4} \\ Area=\frac{15}{4}\cdot\frac{9}{4}=\frac{15\cdot9}{4\cdot4} \\ Area=\frac{135}{16}=8\frac{7}{16} \\ Area=8\frac{7}{16}in^2 \\ \\ \therefore Area=8\frac{7}{16}in^2 \end{gathered}[/tex]

Caleb's recipe calls for 4.4 cups of an ingredient. His measuring bowl only has measurements marked in liters. Caleb knows that one cup is approximately 240 milliliters. Determine the number of liters of that ingredient that Caleb needs for his recipe.

Answers

[tex]\begin{gathered} \text{Caleb's recipe n}eeds\Rightarrow4.4\text{ cups of ingredient} \\ 1\text{ cup is equal to 240 milileters} \\ so,\text{ } \\ 1\text{cup}=240ml=0.24\text{ liter} \\ \text{then,} \\ \Rightarrow4.4cups=4.4\times0.24\text{ liter} \\ \Rightarrow1.056\text{ liter} \end{gathered}[/tex]

Consider a die rolling game that costs $15 per play. Rolling a 6 wins you $40, rolling a 5 gets you$25, and rolling anything else gets you nothing. Find the expected value and determine if the gameis fair.

Answers

Explanation

We need to present the information given in a table as follows:

The second column shows the probability of obtaining each event. Since they are equiprobable events, their probability are 1/6.

The next column determine the cost of obtaining each event; keeping in mid that each play costs $15.

The last column shows the expected value.

We know that If the value of the game is zero i.e. there is no loss or gain for any player, then the game is fair.

According to the table, there is a loss of 25/6, hence, the game is not fair.

Other Questions
your calculation.11. What is the kinetic energy of a 2kg rabbit hopping at a speed of 1.25 m/s? According to the statements provided, name the lines that are parallel to each other. If there is notenough information to conclude parallel, write "not enough info" and explain why. solve following equation6+y=18 Line AB is parallel to line CD. What is the measure of Z1?1/2BA3/45 807/8D what is 2+2?I just cant figure it out. please try to do the work detailed with answers and work. rita Bob and dale served a total of 83 orders Monday at the school cafeteria. rita served 9 fewer orders than bob. Dale served 2 times as many orders as bob. how many orders did they each serve A line passes through point A(2,5) and B(-8,-4). Find the slope of the line. what is the solution set of y equals x squared plus 2X + 7 + y equals x + 7 A mass of 10 kg, initially at rest on a horizontal frictionless surface, is acted upon by a horizontal force of 25 N. The speed of the mass after it has moved 5.0 m is: Can someone please help me out with this Given b(x) = [X+41, what is b(-10)?O-10O -614 maya is an african american student who works hard to speak standard american english when she is at school. when she does this, she is A restaurant is increasing all of its prices by 8%.Write an equation to calculate the new price, n, when the original price is p dollars. Use the figure below to complete the following problem.Given:FLAG isGXYZ isGYZY= In the right triangle ABC angles B and C are congruent. What is the measure of B and C? The land ordinance of _______ determined how many western lands would be __________ How do you find any unit price? please help me and also l will send you the pic What is the quotient and the remainder of 263